GMATPrep - Product of a list of integers

This topic has expert replies
Legendary Member
Posts: 708
Joined: Sun Jun 01, 2008 4:59 am
Location: USA
Thanked: 13 times
Followed by:1 members

GMATPrep - Product of a list of integers

by niraj_a » Sun Jun 01, 2008 5:37 pm
Hello,

I need help with the following DS problem I faced in GMATPrep.

----
A certain list consists of several different integers. Is the product of all the integers positive?

1. The product of the greatest and the smallest of the integers in the list is positive.

2. There is an even number of integers in the list.
----

The OA is C, I picked E (after eliminating A/D/B easily), simply because I though even if the product of two numbers in a 4 number list is positive, even if 1 out of the remaining 2 numbers being negative will make the whole product negative.

Please advise on the chink in my logic. Thanks!

Master | Next Rank: 500 Posts
Posts: 231
Joined: Mon Jan 07, 2008 9:39 am
Thanked: 4 times
Followed by:1 members

by khurram » Sun Jun 01, 2008 6:26 pm
there is a great solution for this somewhere on the forum

basically

least and greatest same means all numbers are either pos or neg

if even then neg becomes positive

Khurram

Legendary Member
Posts: 708
Joined: Sun Jun 01, 2008 4:59 am
Location: USA
Thanked: 13 times
Followed by:1 members

by niraj_a » Sun Jun 01, 2008 6:56 pm
i tried a search but no luck, if anyone can post a link to the explanation i'd really appreciate it.

thanks!

Master | Next Rank: 500 Posts
Posts: 127
Joined: Sat Apr 19, 2008 1:02 am
Thanked: 12 times

Re: GMATPrep - Product of a list of integers

by jasonc » Sun Jun 01, 2008 6:56 pm
niraj_a wrote:Hello,

I need help with the following DS problem I faced in GMATPrep.

----
A certain list consists of several different integers. Is the product of all the integers positive?

1. The product of the greatest and the smallest of the integers in the list is positive.

2. There is an even number of integers in the list.
----

The OA is C, I picked E (after eliminating A/D/B easily), simply because I though even if the product of two numbers in a 4 number list is positive, even if 1 out of the remaining 2 numbers being negative will make the whole product negative.

Please advise on the chink in my logic. Thanks!
The chink in your logic stems from stem1 :)

stem1
we're not looking at the product of any 2 numbers in the list, we're looking at the product of the LARGEST & SMALLEST, meaning if they're both positive, the others CANNOT be negative.
So stem1 tells us the numbers are either all negative or all positive (largest & smallest are the same sign), but we don't know which. If they're all negative and we have an odd number of elements then we would still have a negative product of all elements in the set.

stem2 tells us that there are an even number of elements, which eliminates the uncertainty from stem1
I beat the GMAT! 760 (Q49/V44)

Legendary Member
Posts: 1169
Joined: Sun Jul 06, 2008 2:34 am
Thanked: 25 times
Followed by:1 members

by aj5105 » Sat Oct 04, 2008 6:42 am
nice problem :D

marked E tho..